• 沒有找到結果。

Is f integrable on K? Suppose that f is integrable on K, is it true that R Kf = limR Kfn K? Examples

N/A
N/A
Protected

Academic year: 2022

Share "Is f integrable on K? Suppose that f is integrable on K, is it true that R Kf = limR Kfn K? Examples"

Copied!
11
0
0

加載中.... (立即查看全文)

全文

(1)

Question. Let {fn} be a sequence of integrable functions that converges at every point of a cell K ⊂ Rp to a function f. Is f integrable on K? Suppose that f is integrable on K, is it true that R

Kf = limR

Kfn K?

Examples.

(a) Let Q ∩ [0, 1] = {xn}n=1 and fn be a monotone sequence of integrable functions on [0, 1] defined by fn(x) =

(1 if x ∈ {x1, x2, . . . , xn}, 0 otherwise.

Then the limit function f is defined by f (x) = lim fn(x) = (

1 if x ∈ Q ∩ [0, 1], 0 if x ∈ [0, 1] \ Q.

Note that the convergence of fn to f is not uniform on [0, 1], f is not integrable on [0, 1], and 0 = lim

Z 1

0

fn 6=

Z 1

0

lim fn since R1

0 f does not exist.

(b) Define (discontinuous) fn and (continuous) f on K = [0, 1] by n ≥ 1 by fn(x) =

(

n if x ∈ (0,n1),

0 otherwise, and f (x) = 0.

Note that the convergence of fn to f is not uniform on [0, 1], f is Riemann integrable on K, and 1 = limR1

0 fn 6=R1

0 lim fn=R1

0 f = 0.

(c) Let K = [0, 1], and (continuous function) fn be defined for n ≥ 2 by fn(x) =





n2x if x ∈ [0,n1],

−n2(x −n2) if x ∈ [n1,n2], 0 if x ∈ [n2, 1],

and (continuous) f (x) = lim fn(x) = 0 for all x ∈ K. Note that the convergence of fn to f is not uniform on [0, 1], f is integrable on K, and 1 = limR1

0 fn6=R1

0 lim fn=R1

0 f = 0.

These examples indicate that a convergence theorem for the Riemann integral will require some con- dition in addition to pointwise convergence.

Theorem. Let {fn} be a sequence of integrable functions that converges uniformly on a closed cell K ⊂ Rp to a function f. Then f is integrable and R

Kf = limR

Kfn.

Proof. Let ² > 0 and N be such that kfN − f kK < ². Now let PN be a partition of K such that if P, Q are refinements of PN, then |SP(fN, K) − SQ(fN, K)| < ², for any choice of the intermedi- ate points. If we use the same intermediate points for f and fN, then |SP(fN, K) − SP(f, K)| ≤ kfN−f kKc(K) < ²c(K). Since a similar estimate holds for the partition Q, then for refinements P, Q of PN and corresponding Riemann sums, we have |SP(f, K) − SQ(f, K)| ≤ |SP(f, K) − SP(fN, K)| +

|SP(fN, K) − SQ(fN, K)| + |SQ(fN, K) − SQ(f, K)| ≤ ²(1 + 2c(K)). This implies that f is integrable on K.

Since |R

Kf −R

Kfn| = |R

K(f − fn)| ≤ kf − fnkKc(K), we have R

Kf = limR

Kfn. Example. Let K = [0, 1], and fn be defined by

fn(x) = (

sin(nπx) if x ∈ [0,n1], 0 if x ∈ (n1, 1].

Note that fnconverges to the zero function on [0, 1], and the convergence is not uniform on K. How- ever, limR1

0 fn = lim 2

= 0 = R1

0 lim fn. This example demonstrates that the uniform convergence is not a necessary condition in the theorem.

Bounded Convergence Theorem. Let {fn} be a sequence of integrable functions on a closed cell K ⊂ Rp. Suppose that there exists B > 0 such that kfn(x)k ≤ B for all n ∈ N, x ∈ K. If the function f (x) = lim fn(x), x ∈ K, exists and is integrable, then R

Kf = limR

Kfn.

Remark. This theorem has replaced the uniform convergence of fn by the uniform boundedness of fn and the integrability of f.

(2)

Outline of the Proof. Since f (x) = lim fn(x) for x ∈ K and kfnkK ≤ B for all n ∈ N, there exists M such that |f (x)| ≤ M and |fn(x)| ≤ M for all x ∈ K and all n ≥ 1. Since |f − fn| is integrable on K, there exists a subset A ⊆ K such that c(K \ A) = 0 and |f − fn| converges uniformly to 0 on A. This implies thatR

Kf = limR

Kfn.

To find A, we observe that the convergence of fn to f is not uniform on K if there exists ² > 0 such that the set An= {x ∈ K | ∃ j ≥ n such that |fj(x) − f (x)| ≥ ²} 6= ∅. Note that

(a) {An} is a nested sequence, i.e. A1 ⊇ A2 ⊇ · · · An⊇ An+1 ⊇ · · · ,

(b) lim

n→∞An =

\ n=1

An = ∅ and lim c(An) = 0,

(c) An6= ∅ for any δ < ²,

(d) fn converges uniformly to f on each K \ Aj, j ∈ N.

Examples. Use suitable convergence theorem to prove the following.

(a) If a > 0, then lim

n

Z a

0

e−nxdx = 0.

(b) If 0 < a < 2, then lim

n

Z 2

a

e−nx2dx = 0. What happens if a = 0?

(c) If a > 0, then lim

n

Z π

a

sin nx

nx dx = 0. What happens if a = 0?

(d) Let fn(x) = nx

1 + nx for x ∈ [0, 1], and let f (x) =

(0 if x = 0, 1 if x ∈ (0, 1].

Then lim fn(x) = f (x) for all x ∈ [0, 1] and that limR1

0 fn(x) dx = R1

0 f (x) dx.

(e) Let hn(x) = nxe−nx2 for x ∈ [0, 1] and let h(x) = 0. Then 0 =R1

0 h(x) dx 6= limR1

0 hn(x) dx = 1 2. Monotone Convergence Theorem. Let {fn} be a monotone sequence of integrable functions on a closed cell K ⊂ Rp. If the function f (x) = lim fn(x), x ∈ K, exists and is integrable, then R

Kf = limR

Kfn.

Proof. Suppose that f1(x) ≤ f2(x) ≤ · · · ≤ f (x) for all x ∈ K, then fn(x) ∈ [f1(x), f (x)] for all n ∈ N and kfn(x)k ≤ |f1(x)| + |f (x)| ≤ sup

x∈K

|f1(x)| + sup

x∈K

|f (x)| = B for all x ∈ K and for all n ∈ N, so we can apply the Bounded Convergence Theorem to establish that R

Kf = limR

Kfn. Remark. Note that the convergence theorem may fail if K in not compact.

Example. Let fn(x) =



 1

x if x ∈ [1, n]

0 if x > n. Then each fn is integrable on [1, ∞), and {fn} is a bounded, monotone sequence that converges uniformly on [1, ∞) to a continuous function f (x) = 1/x.

Note that limR

1 fn6=R

1 lim fn since f is not integrable on [1, ∞).

Example. Let gn(x) =



 1

n if x ∈ [0, n2]

0 if x > n2. Then each gn is integrable on [1, ∞), and {gn} is a bounded, monotone sequence that converges [1, ∞) to an integrable function g(x) ≡ 0. Note that limR

1 gn6=R

1 lim gn.

Definition. If {fn} is a sequence of functions defined on a subset D of Rp with values in Rq, the sequence of partial sums (sn) of the series P

fn is defined for x in D by sn(x) = Xn

j=1

fj(x). In case the sequence {sn} converges on D to a function f, we say that the infinite series of functionsP

fn

(3)

converges to f on D. If the sequence {sn}converges uniformly on D to a function f, we say that the infinite series of functionsP

fn converges uniformly to f on D.

Remark (Cauchy Criterion). It is easy to see that P

fk converges uniformly on D if and only if for each ² > 0, there exists M = M(²) ∈ N such that for any n, m ≥ M and any x ∈ D, we have ksn(x) − sm(x)k < ².

Dirichlet’s Test Let {fn} be a sequence of functions on D ⊆ Rp to Rq such that the partial sums sn=

Xn j=1

fj, n ∈ N, are all bounded. Let {φn} be a decreasing sequence of functions on D to R which

converges uniformly on D to zero. Then the series X n=1

(fnφn) converges uniformly on D.

Abel’s Test Let X n=1

fn be a series of functions on D ⊆ Rp to Rq which is uniformly convergent on

D. Let {φn} be a sequence of functions on D to R which is bounded on D. Then the series X n=1

(fnφn) converges uniformly on D.

Outline of the proofs. For Dirichlet’s test, observe that | Xm j=n

φjfj| = |φm+1sm− φnsn−1+ Xm j=n

j

φj+1sj|. For Abel’s test, if |φj(x)| ≤ B for all j ∈ N and for all x ∈ D, then | Xm j=n

φjfj| ≤ B Xm j=n

|fj|.

Theorem. If fn is continuous on D ⊆ Rp to Rq for each n ∈ N and if P

fn converges to f uniformly on D, then f is continuous on D.

Term-by-Term Integration Theorem. Suppose that the real-valued functions fn, n ∈ N, are integrable on K = [a, b]. If the series P

fn converges to f uniformly on K, then f is integrable on K and

Z

K

f = X

j=1

Z

K

fn.

Term-by-Term Differentiation Theorem. For each n ∈ N, let fn be a real-valued function on K = [a, b] which has a derivative fn0 on K. Suppose that the infinite series P

fn converges for at least one point of K and that the series of derivatives P

fn0 converges uniformly on K. Then there exists a real-valued function f on K such that P

fn converges uniformly on K to f. In addition, f has a derivative on K and f0 =P

fn0.

Proof. Suppose that the partial sum snofP

fnconverges at x0 ∈ K. For each x ∈ K and any m, n ∈ N, by the Mean Value Theorem, the equality sm(x)−sn(x) = sm(x0)−sn(x0)+(x−x0)(s0m(y)−s0n(y)) holds for some y lying between x and x0. The uniform convergence of P

fn0 and the convergence of Pfn(x0) lead to the uniform convergence of P

fn on K.

Suppose thatP

fn0 converges uniformly to g on K. For each x, c ∈ K and any m, n ∈ N, by the Mean Value Theorem, the equality sm(x) − sn(x) = sm(c) − sn(c) + (x − c)(s0m(y) − s0n(y)) holds for some y lying between x and c. We infer that, when x 6= c, then |sm(x) − sm(c)

x − c −sn(x) − sn(c)

x − c | ≤ ks0m−s0nkK. Given ² > 0, by the uniform convergence ofP

fn0, there exists a M(²) such that if m, n ≥ M(²) then ks0m− s0nkK < ². Taking the limit with respect to m, we get |f (x) − f (c)

x − c −sn(x) − sn(c)

x − c | ≤ ² when n ≥ M(²). Since g(c) = lim s0n(c), there exists an N(²) such that if n ≥ N(²), then |s0n(c) − g(c)| < ².

Now let L = max{M(²), N(²)}. In view of the existence of s0L(c), if 0 < |x − c| < δL(²), then

|sL(x) − sL(c)

x − c −s0L(c)| < ². Therefore, it follows that if 0 < |x−c| < δL(²), then |f (x) − f (c)

x − c −g(c)| <

3². This shows that f0(c) exists and equals g(c).

Example (a). For each k ∈ N and for each x ∈ [−1, 1], define fk(x) = xk

k2. Then X k=1

fk converges

(4)

uniformly on [−1, 1], and X

k=1

fk0 = X k=1

xk−1

k converges uniformly on any [−r, r], where 0 ≤ r < 1.

Example (b). For each k ≥ 0 and for each x ∈ (−1, 1), define fk(x) = (−1)kxk. Then X k=0

fk converges to f (x) = 1

1 + x uniformly on any [−r, r], where 0 ≤ r < 1.

Example (c). Using (a), one observes that X

k=0

fk0 = X k=0

(−1)kx2k converges to 1

1 + x2 on (−1, 1) and it is not convergent at x = ±1, while

X k=0

fk = X k=0

(−1)k

2k + 1x2k+1 converges to tan−1x on (−1, 1) uniformly on [−1, 1].

Example (d). Note that X k=1

fk = X k=1

sin kx

k2 converges uniformly on R by Cauchy Criterion and the criterion is not applicable for

X k=1

fk0 = X k=1

cos kx k since

X k=1

1

k diverges.

Definition. Let f be defined on [a, ∞) × [α, β] to R. Suppose that for each t ∈ J = [α, β] the infinite integral F (t) = R

a f (x, t)dx = limc→∞Rc

af (x, t)dx exists. We say that this convergence is uniform on J if for every ² > 0 there exists a number M(²) such that if c ≥ M(²) and t ∈ J, then

|F (t) −Rc

a f (x, t)dx| < ².

Dominated Convergence Theorem. Suppose that f is integrable over [a, c] for all c ≥ a and all t ∈ J = [α, β]. Suppose that there exists a positive function φ defined for x ≥ a such that

|f (x, t)| ≤ φ(x) for x ≥ a, t ∈ J, and such that the integral R

a φ(x)dx exists. Then, for each t ∈ J, the integral F (t) =R

a f (x, t)dx is (absolutely) convergent and the convergence is uniform on J.

Dirichlet’s Test Let f be continuous on [a, ∞) × [α, β] and suppose that there exists a constant A such that |Rc

af (x, t)dx| ≤ A for c ≥ a, t ∈ J = [α, β]. Suppose that for each t ∈ J, the function φ(x, t) is monotone decreasing for x ≥ a, and converges to 0 as x → ∞ uniformly for t ∈ J. Then the integral F (t) =R

a f (x, t)φ(x, t)dx converges uniformly on J.

Examples.

(a) R

0

dx

x2+ t2 converges uniformly for |t| ≥ a > 0.

(b) R

0

dx

x2+ t converges uniformly for t ≥ a > 0 and diverges when t ≤ 0.

(c) R

0 e−xcos txdx converges uniformly for t ∈ R by the dominated convergence theorem.

(d) R

0 e−x2−t2/x2dx converges uniformly for t ∈ R by the dominated convergence theorem.

Theorem. Let f be continuous on K = [a, b] × [c, d] to R and F : [c, d] → R be defined by F (t) =Rb

af (x, t)dx. Then F is continuous on [c, d] to R.

Proof. Let ² > 0, since f is uniformly continuous on K, there exists a δ(²) > 0 such that if t and t0 belong to [c, d] and |t − t0| < δ(²), then |f (x, t) − f (x, t0)| < ², for all x ∈ [a, b]. It follows that

|F (t) − F (t0)| = |Rb

a

¡f (x, t) − f (x, t0

dx| ≤ Rb

a|f (x, t) − f (x, t0)|dx ≤ ²(b − a), which establishes the continuity of F.

Remark. Suppose that f is continuous on [a, ∞) × [c, d] to R and F (t) = R

a f (x, t)dx converges uniformly on [c, d], we let Fn(t) =Ra+n

a f (x, t)dx. Then Fnis continuous on [c, d] and F is continuous on [c, d] since Fn converges to F uniformly on [c, d].

Theorem. Let f and its partial derivative ft be continuous on K = [a, b] × [c, d] to R. Then the function F (t) =Rb

a f (x, t)dx is differentiable on (c, d) and F0(t) =Rb

a ft(x, t)dx for t ∈ (c, d).

Proof. From the uniform continuity of ft on K we infer that if ² > 0, then there is a δ(²) > 0 such

(5)

that if |t − t0| < δ(²), then |ft(x, t) − ft(x, t0)| < ² for all x ∈ [a, b]. Let t, t0 satisfy this condition and apply the Mean Value Theorem to obtain a t1 ( which may depend on x and lies between t and t0) such that f (x, t) − f (x, t0) = (t − t0)ft(x, t1). Combining these two relations, we infer that if 0 < |t − t0| < δ(²), then |f (x, t) − f (x, t0)

t − t0 − ft(x, t0)| < ² for all x ∈ [a, b]. Thus, we obtain

|F (t) − F (t0) t − t0 Rb

a ft(x, t0)dx| ≤ Rb

a |f (x, t) − f (x, t0)

t − t0 − ft(x, t0)|dx ≤ ²(b − a), which establishes F0(t) =Rb

a ft(x, t)dx.

Generalization. Let S be a measurable subset of Rn and T a subset of Rm. Suppose f (x, y) is a function on T × S that is integrable as a function of y ∈ S for each x ∈ T, and let F be defined on T by F (x) =R

Sf (x, y)dy for x ∈ T.

(a) If f (x, y) is continuous as a function of x ∈ T for each y ∈ S, and there exists a constant C such that |f (x, y)| ≤ C for all x ∈ T and y ∈ S, then F is continuous on T.

(b) Suppose T is open. If f (x, y) is of class C1 as a function of x ∈ T for each y ∈ S, and there is a constant C such that |∇xf (x, y)| ≤ C for all x ∈ T and y ∈ S, then F is of class C1 on T and

∂F

∂xj

(x) =R

S

∂f

∂xj

(x, y)dy for x ∈ T.

Proof. Let {xj} be a sequence in T converging to x ∈ T. For each j ∈ N and y ∈ S, let fj(y) = f (xj, y) and let f (y) = f (x, y). Then each fj and f are integrable on S, and |fj(y)| ≤ C and fj(y) → f (y) for all j and all y ∈ S. The bounded convergence theorem implies that lim F (xj) = limR

Sf (xj, y)dy = limR

Sfj = R

Slim fj = R

Slim f (xj, y) = R

Sf (x, y) = F (x). Hence, F is continu- ous and this proves (a).

Part (b) is proved by applying the bounded convergence theorem to the sequence of difference quo- tients f (x + hjei, y) − f (x, y)

hj , where ei denotes the unit vector in the xi−coordinate and {hj} is a sequence of numbers tending to zero. The uniform bound on these quotients is obtained by applying the mean value theorem .

Examples.

(a) Let f (x, t) = cos tx

1 + x2 for x ∈ [0, ∞) and t ∈ (−∞, ∞). Then R

0 f (x, t) converges uniformly for t ∈ R by Dominated Convergence Theorem.

(b) Let f (x, t) = e−xxt for x ∈ [0, ∞) and t ∈ [0, ∞). For any β > 0, the integral R

0 f (x, t) converges uniformly for t ∈ [0, β] by Dominated Convergence Theorem. Similarly, the Laplace transform of xn, n = 0, 1, 2, . . . , defined by L {xn}(t) = R

0 xne−txdx also converges uniformly for t ≥ γ > 0 to n!

tn+1. For t ≥ 1, define the gamma function Γ by Γ(t) = R

0 xt−1e−xdx. Then it is uniformly convergent on an interval containing t. Note that Γ(t + 1) = tΓ(t) and hence Γ(n + 1) = n! for any n ∈ N.

(c) Let f (x, t) = e−txsin x for x ∈ [0, ∞) and t ≥ γ > 0. Then the integral F (t) = R

0 e−txsin xdx is converges uniformly for t ≥ γ > 0 by Dominated Convergence Theorem and it is called the laplace transform of sin x, denoted by L {sin x}(t). Note that an elementary calculation shows that L {sin x}(t) = 1

1 + t2. (d) Let f (x, t, u) = e−txsin ux

x for x ∈ [0, ∞) and t, u ∈ [0, ∞). By taking φ = e−tx/x and by applying the Dirichlet’s test, one can show that R

γ f (x, t, u) converges uniformly for t ≥ γ ≥ 0. Note that if F (t, u) = L {sin ux

x }(t) =R

0 e−txsin ux

x dx, then ∂F

∂u(t, u) =R

0 e−txcos uxdx = t t2+ u2 and F (t, u) = tan−1 u

t. By setting u = 1 and by letting t → 0+, we obtain thatR

0

sin x

x dx = π 2.

(6)

(e) Let G(t) =R

0 e−x2−t2/x2dx for t > 0. Then G0(t) = −2G(t) and G(t) =

√π 2 e−2t. (f) Let F (t) =R

0 e−x2cos txdx for t ∈ R. Then F0(t) = −t

2F (t) and F (t) =

√π 2 e−t2/4.

Leibiniz’s Formula. Let f and its partial derivative ft be continuous on K = [a, b] × [c, d] to R and α and β be differentiable functions on [c, d] and have values in [a, b]. Then the function φ(t) = Rβ(t)

α(t) f (x, t)dx is differentiable on (c, d) and φ0(t) = f (β(t), t)β0(t) − f (α(t), t)α0(t) +Rβ(t)

α(t) ft(x, t)dx for t ∈ (c, d).

Proof. Let H be defined for (u, v, t) by H(u, v, t) = Ru

v f (x, t)dx, where u, v belong to [a, b] and t belongs to [c, d]. Then φ(t) = H(β(t), α(t), t). Applying the Chain Rule to obtain the result.

Interchange Theorem. Let f be continuous on K = [a, b] × [c, d] to R.

Then Rd

c

nRb

a f (x, t)dx o

dt =Rb

a

nRd

c f (x, t)dt o

dx.

Proof. Since f is uniformly continuous on K, if ² > 0 there exists a δ(²) > 0 such that if |x−x0| < δ(²) and |t − t0| < δ(²), then |f (x, t) − f (x0, t0)| < ². Let n be chosen so large that (b − a)/n < δ(²) and (d − c)/n < δ(²) and divide K into n2 equal rectangles by dividing [a, b] and [c, d] each into n equal parts. For j = 0, 1, . . . , n, we let xj = a + (b − a)j/n, tj = c + (d − c)j/n.

Then Z d

c

½Z b

a

f (x, t)dx

¾ dt =

Xn i=1

Xn j=1

Z ti

ti−1

(Z xj

xj−1

f (x, t)dx )

dt = Xn

i=1

Xn j=1

f (x0j, t0i)(xj − xj−1)(ti ti−1).

Similarly, Z b

a

½Z d

c

f (x, t)dt

¾ dx =

Xn i=1

Xn j=1

f (x00j, t00i)(xj− xj−1)(ti− ti−1).

The uniform continuity of f implies that two iterated integrals differ by at most ²(b − a)(d − c). Since

² > 0 is arbitrary, the equality of these integrals is confirmed.

Example. Let A ⊆ R2 be the set consisting of all pairs (i/p, j/p) where p is a prime number, and i, j = 1, 2, . . . , p − 1. (a) Show that each horizontal and each vertical line in R2 intersects A in a finite number (often zero) of points and that A does not have content. Let f be defined on K = [0, 1]×[0, 1]

by f (x, y) = 1 for (x, y) ∈ A and f (x, y) = 0 otherwise. (b) Show that f is not integrable on K.

However, the iterated integrals exist and satisfyR1

0

nR1

0 f (x, y)dx o

dy =R1

0

nR1

0 f (x, y)dy o

dx.

Example. Let K = [0, 1] × [0, 1] and let f : K → R be defined by f (x, y) =



0 if either x or y is irrational, 1

n if y is rational and x = m

n where m and n > 0 are relatively prime integers.

Show thatR

Kf =R1

0

nR1

0 f (x, y)dxo

dy = 0, but that R1

0 f (x, y)dy does not exist for rational x.

Fubini’s Theorem. Let f be continuous on K = [a, b]×[c, d] to R. ThenR

Kf =Rd

c

nRb

af (x, y)dx o

dy = Rb

a

nRd

c f (x, y)dy o

dx.

Proof. Let F be defined for y ∈ [c, d] by F (y) = Rb

af (x, y)dx. Let c = y0 < y1 < · · · < yr = d be a partition of [c, d], let a = x0 < x1 < · · · < xs = b be a partition of [a, b], and let P denote the partition of K obtained by using the cells [xi−1, xi] × [yj−1, yj]. Let yj be any point in [yj−1, yj] and note that F (yj) =

Z b

a

f (x, yj)dx = Xs

i=1

Z xi

xi−1

f (x, yj)dx. The Mean Value Theorem implies that

for each j, there exists a xji ∈ [xi−1, xi] such that F (yj) = Xs

i=1

f (xji, yj)(xi− xi−1). We multiply by

(yj− yj−1) and add to obtain Xr

j=1

F (yj)(yj− yj−1) = Xr

j=1

Xs i=1

f (xji, yj)(xi− xi−1)(yj− yj−1). We have shown that an arbitrary Riemann sum for F on [c, d] is equal to a particular Riemann sum of f on

(7)

K corresponding to the partition P. Since f is integrable on K, the existence of the iterated integral and its equality with the integral on K is established.

A minor modification of the proof given for the preceding theorem yields the following, slightly stronger, assertion.

Generalization Theorem. Let f be integrable on K = [a, b] × [c, d] to R and suppose that for each y ∈ [c, d], the function x 7→ f (x, y) of [a, b] into R is continuous except possibly for a finite number of points, at which it has one-sided limits. Then R

Kf = Rd

c

nRb

a f (x, y)dx o

dy.

Corollary. Let A ⊆ R2 be given by A = {(x, y) : α(y) ≤ x ≤ β(y), c ≤ y ≤ d}, where α and β are continuous functions on [c, d] with values in [a, b]. If f is continuous on A 7→ R, then f is integrable on A andR

Af =Rd

c

nRβ(y)

α(y) f (x, y)dx o

dy.

Proof. Let K ⊇ A be a closed cell and fK be the extension of f to K. Since ∂A has content zero, fK is integrable on K. Now for each y ∈ [c, d] the function x 7→ fK(x, y) is continuous except possibly at the two points α(y) and β(y), at which it has one-sided limits. It follows from the preceding theorem that R

Af =R

KfK =Rd

c

nRb

afK(x, y)dx o

dy =Rd

c

nRβ(y)

α(y) f (x, y)dx o

dy.

Example. Let R denote the triangular region in the first quadrant bounded by the lines y = x, y = 0, and x = 1. ThenR1

0

R1

y

sin x

x dxdy =R1

0

Rx

0

sin x x dydx.

Example.

Z 2

0

Z 1

y/2

ye−x3 dxdy = Z 1

0

Z 2x

0

ye−x3 dydx = Z 1

0

y2

2 e−x3|2x0 dx

= Z 1

0

2x2e−x3dx = −2e−x3 3 |10 = 2

3(1 − e−1).

Example. For β > α > 0, let R = [0, ∞) × [α, β] and f (x, t) = e−tx. Then log β

α = Rβ

α

1 tdt = Rβ

α

R

0 e−txdxdt =R

0

Rβ

α e−txdtdx =R

0

e−αx− e−βx

x dx.

Lemma. Let Ω ⊆ Rp be open, φ : Ω → Rp belong to Class C1(Ω), and A be a bounded set with Cl(A) = ¯A ⊂ Ω.

Then there exists a bounded open set Ω1 with

A ⊂ Ω¯ 1 ⊂ ¯1 ⊆ Ω

and a constant M > 0 such that if A is contained in the union of a finite number of closed cells in Ω, with total content at most α, then φ(A) is contained in the union of a finite number of closed cells in Ω, with total content at most (√

pM )pα.

Proof. If Ω = Rp, let δ = 1; otherwise let δ = 1

2inf{ka − xk : a ∈ ¯A, x /∈ Ω} > 0.

Let

1 = {y ∈ Rp : ky − ak < δ for some a ∈ A}

M = sup{kdφ(x)kpp= sup

06=v∈Rp

kdφx(v)k/kvk : x ∈ Ω1} < ∞.

If A ⊆ I1∪ · · · ∪ Im, where the Ij are closed cells contained in Ω1, then it follows that for x, y ∈ Ij we have

kφ(x) − φ(y)k ≤ Mkx − yk.

Suppose the side length of Ij is 2rj and take x to be the center of Ij; then if y ∈ Ij, we have kx − yk ≤√

prj. Thus φ(Ij) is contained in a closed cell of side length 2

pMrj, and φ(A) is contained in the union of a finite number closed cells with total content at most (

pM )pα.

Theorem. Let Ω ⊆ Rp be open, φ : Ω → Rp belong to Class C1(Ω). If A ⊂ Ω has content zero and if ¯A ⊂ Ω, then φ(A) has content zero.

(8)

Proof. Apply the lemma for arbitrary α > 0.

Corollary. Let r < p, Ω ⊆ Rr be open, and ψ : Ω → Rp belong to Class C1(Ω). If A ⊂ Ω is a bounded set with ¯A ⊂ Ω, then ψ(A) has content zero in Rp.

Proof. Let Ω0 = Ω × Rp−r. Then Ω0 is open in Rp. Define φ : Ω0 → Rp by

φ(x1, . . . , xr, xr+1, . . . , xp) = ψ(x1, . . . , xr).

Thus φ ∈ C1(Ω0).

Let A0 = A × {(0, . . . , 0)}. Then ¯A0 ⊂ Ω0 and A0 has content zero in Rp. It follows that ψ(A) = φ(A0) has content zero in Rp.

Theorem. Let Ω ⊆ Rp be open, φ : Ω → Rp belong to Class C1(Ω). Suppose that A has content, A ⊂ Ω, and the Jacobian determinant J¯ φ(x) = det(dφ)(x) 6= 0 for all x ∈ Int(A). Then φ(A) has content.

Proof. Since φ( ¯A) is compact and φ(A) ⊆ φ( ¯A), φ(A) is bounded in Rp, and φ(A) ⊆ φ( ¯A).

Now ∂φ(A) ∪ Int(φ(A)) = φ(A) ⊆ φ( ¯A) = φ(Int(A) ∪ ∂A),

and since φ ∈ C1(Ω) and Jφ(x) 6= 0 for all x ∈ Int(A), we conclude that φ(Int(A)) ⊆ Int(φ(A)) by the inverse function theorem.

Hence, we infer that ∂φ(A) ⊆ φ(∂A), and c(∂φ(A)) = 0.

Thus φ(A) has content.

Corollary. Let Ω ⊆ Rp be open, φ : Ω → Rp belong to Class C1(Ω) and be injective on Ω.

If A has content, ¯A ⊂ Ω, and Jφ(x) 6= 0 for all x ∈ Int(A).

Then ∂φ(A) = φ(∂A).

Proof. The proof of the inclusion φ(∂A) ⊇ ∂φ(A) is given in the proof of the proceeding theorem.

To establish the identity ∂φ(A) = φ(∂A), we only need to show the that φ(∂A) ⊆ ∂φ(A).

Let x ∈ ∂A, then there exists a sequence {xn} in A and a sequence {yn} in Ω \ A, such that

n→∞lim xn= x = lim

n→∞yn.

Since φ is continuous, we have lim

n→∞φ(xn) = φ(x) = lim

n→∞φ(yn).

On the other hand, since φ is injective on Ω, φ(yn) /∈ φ(A).

Thus φ(x) ∈ ∂φ(A) which implies that φ(∂A) ⊆ ∂φ(A).

Theorem. Let

L ∈ L (Rp) = {L : Rp → Rp | L = (lij) is an p × p matrix over R}

= the space of linear mappings on Rp, and let A ∈ D(Rp).

Then c(L(A)) = | det L| c(A).

Outline of the Proof. If det L = 0, then L(Rp) = Rr for some r < p, and that c(L(A)) = 0 for all A ∈ D(Rp).

If det L 6= 0, and if A ∈ D(Rp), then L(A) ∈ D(Rp).

Also, note that

(1) if A ∈ D(Rp), then L(A) has content and c(L(A)) ≥ 0.

(2) suppose A, B ∈ D(Rp) and A∩B = ∅, then L(A)∩L(B) = ∅ and c(L(A∪B)) = c(L(A)∪L(B)) = c(L(A)) + c(L(B)).

(3) if x ∈ Rp and A ∈ D(Rp), then L(x + A) = L(x) + L(A) and c(L(x + A)) = c(L(A)).

These imply that

(i) there exists a constant mL ≥ 0 such that c(L(A)) = mLc(A) for all A ∈ D(Rp).

(9)

(ii) if L, M ∈ L (Rp) are nonsingular maps and if A ∈ D(Rp), since mL◦Mc(A) = c(L ◦ M(A)) = mLc(M(A)) = mLmMc(A), we have mL◦M = mLmM.

(iii) one can show that mL= | det L| since every nonsingular L ∈ L (Rp) is the composition of linear maps of the following three forms:

(a) L1(x1, . . . , xp) = (αx1, x2, . . . , xp) for some α 6= 0;

(b) L2(x1, . . . , xi, xi+1, . . . , xp) = (x1, . . . , xi+1, xi, . . . , xp);

(c) L3(x1, . . . , xp) = (x1+ x2, x2, . . . , xp).

If K0 = [0, 1) × · · · × [0, 1) in Rp, then one can show that

mL1 = mL1c(K0) = c(L1(K0)) = |α| = | det L1|, mL2 = mL2c(K0) = c(L2(K0)) = 1 = | det L2|, mL3 = mL3c(K0) = c(L3(K0)) = 1 = | det L3|.

Hence, we have mL= | det L|.

Lemma. Let K ⊂ Rp be a closed cell with center 0, Ω be an open set containing K and ψ : Ω → Rp belong to Class C1(Ω) and be injective. Suppose further that Jψ(x) 6= 0 for x ∈ K and that kψ(x) − xk ≤ αkxk for x ∈ K, and some constant 0 < α < 1

√p. Then

(1 − α√

p)p c(ψ(K))

c(K) ≤ (1 + α√ p)p. Proof. If the side length of K is 2r and if x ∈ ∂K, then we have

r ≤ kxk ≤ r√ p.

This implies that

kψ(x) − xk ≤ αkxk ≤ αr√ p, i.e. ψ(x) is within distance αr√

p of x ∈ ∂K.

Note that Rp\ ∂K is a disjoint union of two nonempty open sets. Let Ci be an open cell with center 0 and side length 2(1 − α√

p)r, Co be a closed cell with center 0 and side length 2(1 + α√

p)r.

Then we have Ci ⊂ ψ(K) ⊂ Co, which implies that (1−α√

p)pc(K) = (1−α√

p)p(2r)p = c(Ci) ≤ c(ψ(K)) ≤ c(Co) = (1+α√

p)p(2r)p = (1+α√

p)pc(K).

The Jacobian Theorem. Let Ω ⊆ Rp be open, φ : Ω → Rp belong to Class C1(Ω), and be injective on Ω with Jφ(x) 6= 0 for x ∈ Ω. Let A have content and satisfy that ¯A ⊂ Ω. If ² > 0 is given, then there exists γ > 0 such that if K is a closed cell with center x ∈ A and side length less than 2γ, then

|Jφ(x)|(1 − ²)p c(φ(K))

c(K) ≤ |Jφ(x)|(1 + ²)p. Proof. For each x ∈ Ω, let Lx = (dφ(x))−1,

then 1 = det(Lx◦ dφ(x)) = (det Lx) (det dφ(x)), it follows that det Lx = 1

det dφ(x) = 1 Jφ(x). Let Ω1 be a bounded open subset of Ω such that

A ⊂ Ω¯ 1 ⊂ ¯1 ⊆ Ω

(10)

and dist(A, ∂Ω1) = 2δ > 0.

Since φ ∈ C1(Ω), the entries in the standard matrix for Lx are continuous There exists a constant M > 0 such that kLxkpp ≤ M for all x ∈ Ω1. Let 0 < ² < 1 be a constant.

Since dφ is uniformly continuous on Ω1, there exists β with 0 < β < δ such that if x1, x2 ∈ Ω1 and kx1− x2k ≤ β, then kdφ(x1) − dφ(x2)kpp ≤ ²/M√

p.

Now let x ∈ A be given, hence if kzk ≤ β, then x, x + z ∈ Ω1. Hence,

kφ(x + z) − φ(x) − dφ(x)(z)k ≤ kzk sup

0≤t≤1kdφ(x + tz) − dφ(x)kpp

²

M√ pkzk.

This implies that for a fixed x ∈ A and for each kzk ≤ β if we set ψ(z) = Lx[φ(x + z) − φ(x)], Then we have

kψ(z) − zk = kLx[φ(x + z) − φ(x) − dφ(x)(z)]k

≤ Mkzk sup

0≤t≤1

kdφ(x + tz) − dφ(x)kpp

²

√pkzk for kzk ≤ β.

If K1 is any closed cell with center 0 and contained in the open ball with radius β, then (1 − ²)p c(ψ(K1))

c(K1) ≤ (1 + ²)p.

It follows that if K = x + K1 then K is a closed cell with center x and that c(K) = c(K1) and c(ψ(K1)) = | det Lx|c(φ(x + K1) − φ(x)) = 1

Jφ(x)c(φ(K)).

This establishes the inequality

|Jφ(x)|(1 − ²)p c(φ(K))

c(K) ≤ |Jφ(x)|(1 + ²)p.

for those closed cell K with center x ∈ A and side length less than 2γ = 2β/√ p.

Change of Variables Theorem. Let Ω ⊆ Rp be open, φ : Ω → Rp belong to Class C1(Ω), and be injective on Ω, and Jφ(x) 6= 0 for x ∈ Ω. Suppose that A has content, ¯A ⊂ Ω, and f : φ(A) → R is bounded and continuous.

Then R

φ(A)f =R

A(f ◦ φ)|Jφ|.

Example. Find RR

S

¡x2 + y2¢

dA if S be the region in the first quadrant bounded by the curves xy = 1, xy = 3, x2− y2 = 1, and x2− y2 = 4. Setting u = x2− y2, v = xy, we haveRR

S

¡x2+ y2¢ Rv=3 dA =

v=1

Ru=4

u=1

1

2 dudv = 3.

Example. ³R

0 e−x2dx´2

=R

0

R

0 e−x2−y2dxdy =Rπ/2

0

R

0 re−r2drdθ = π 2. Summary.

(a) Let A ⊂ Rn. Define what it means to say that A has content (or A is Jordan measurable).

(11)

(b) Let A ⊂ Rn. Define the content c(A) of A when A has content (or A is Jordan measurable).

(c) Let A ⊂ Rn. Define what it means to say that A has content (or measure) c(A) zero.

(d) Let A be a bounded subset of Rn, and let f be a bounded function defined on A to R. Define what it means to say that f is integrable on A. Give a class of A and a class of f from which R

Af exists.

(e) Let A be a bounded subset of Rnand let f be an integrable function defined on A to R. Discuss the continuity of f on A.

(f) Let A be a bounded subset of Rn and let f be a continuous function defined on A to R. Discuss the integrability of f on A?

(g) Let A be a bounded subset of Rn, let f, g be integrable functions defined on A to R, and let a, b ∈ R. Discuss the integrability of af + bg and f g on A.

(h) Let A be a bounded subset of Rn and let {fn} be a sequence of integrable function defined on A to R. Assume that f (x) = lim fn(x) exists for each x ∈ A. Discuss the integrability of f on A, and conditions on which the equality limR

Afn =R

Alim fn holds.

(i) Let Ω ⊆ Rp be open and let φ : Ω → Rp belong to Class C1(Ω). Suppose that A has content (or A is measurable), ¯A ⊂ Ω. Discuss the measurability of φ(A).

(j) Let Ω ⊆ Rp be open and let φ : Ω → Rp belong to Class C1(Ω). Suppose that A has content zero and if ¯A ⊂ Ω, discuss the measurability of φ(A).

(k) Let L ∈ L (Rp) = {B : Rp → Rp | B = (bij) is an p × p matrix over R} = the space of linear mappings on Rp, and let A ∈ D(Rp). Find c(L(A)).

(l) Let Ω ⊆ Rp be open and let φ : Ω → Rp belong to Class C1(Ω). Then dφ(x) : Rp → Rp is a linear mapping in L (Rp). If Jφ(x) 6= 0 for all x ∈ Ω and suppose that A has content (or A is measurable), ¯A ⊂ Ω. Discuss the geometric meaning of dφ(x)(v) for each v ∈ Rp, and the influence of dφ(x) on the volume element of A at x.

參考文獻

相關文件

Calculus Riemann integrable functions (Continued) November 6, 2018. Proof Suppose that f is not uniformly continuous

The idea of the above proof can be applied to the proof of Stoke’s Theorem for general cases... One can show that the Stoke’s theorem holds for any k-form and for any k-chains on R

[r]

By the Lebesgue’s theorem we know that if f is (Riemann) integrable on A, then the discontinuities of f

Determine whether the following sequence of functions {f n } converges pointwise on the indicated interval2. (a) Suppose that {f n } is a sequence of bounded (not

(Notice that β is linearly indepen- dent over R... Part III: Gamma and

Suppose that E is bounded, open interval and that each f k.. is differentiable

Generalization Theorem Let f be integrable on K = [a, b] × [c, d] to R and suppose that for each y ∈ [c, d], the function x 7→ f (x, y) of [a, b] into R is continuous except